Find the height of the triangle by applying formulas for the area of a triangle and your knowledge about triangles.
A. 10.5 cm
B. 3.4 cm
C. 8.5 cm
D. 12 cm

Find The Height Of The Triangle By Applying Formulas For The Area Of A Triangle And Your Knowledge About

Answers

Answer 1

Answer:

12 cm is the right answer pls mark me brainliest

Answer 2

The height of the triangle by applying formulas for the area of a triangle and your knowledge about triangles is 12 cm.

What is Area of Triangle?

The area of a triangle is defined as the total space occupied by the three sides of a triangle in a 2-dimensional plane. The basic formula for the area of a triangle is equal to half the product of its base and height, i.e., A = 1/2 × b × h.

What is Heron's formula?

Heron's formula, formula credited to Heron of Alexandria (c. 62 ce) for finding the area of a triangle in terms of the lengths of its sides. In symbols, if a, b, and c are the lengths of the sides:

Area = √s(s - a)(s - b)(s - c) where s is half the perimeter, or (a + b + c)/2.

Given:

Three sides are: 15cm, 25 cm and 2 cm

Now, Using Heron's formula

semi-perimeter= (25+ 20 + 15)/2

s= 30 cm

Now,

Area of triangle

=√s(s-a)(s-b)(s-c)

=√30* 5 * 10* 15

=√5*2*3*5*2*5*3*5

=5*5*2*3

=150 cm²

Again, area of triangle= 1/2* b* h

150= 1/2* 25* x

12cm= x

Learn more about Area of Triangle here:

https://brainly.com/question/9817285

#SPJ2


Related Questions

x/t+m=b need to make x the subject

Answers

Answer:

x=(t+m)/b is the answer

Step-by-step explanation:

Hope it will help :)

Answer:

x =  t(b-m)

Step-by-step explanation:

x/t + m =b

subtract m from each side

x/t +m-m = b-m

x/t =b-m

Multiply each side by t

x/t *t = t(b-m)

x =  t(b-m)

kelly used 2.1 x 10^6 KB of data so far this month. her younger brother joseph used 7 x 10^5 KB of the data so far this month if their share family plan allows them to use 3,000,000 KB of the data each month, how much data usage o they have available for the remainder of this month?

Answers

Answer:

They will 200,000 kb left

Step-by-step explanation:

  2.1 x 10^6=2,100,000     7 x 10^5=700,000

HELP PLEASE!!
Which best describes the relationship between the lines
with equations -3x – 4y = 1 and -6x – 8y = 2?
A. Perpendicular
B. Neither perpendicular or parallel
C. Parallel
D. Same line

Answers

Answer:

same line

Step-by-step explanation:

its the exact same equation, but the second is multiplied by 2

How to do this question plz answer me step by step plzz ​

Answers

Answer:

  4 cm

Step-by-step explanation:

At 12 cm depth, the depth is 12/15 = 4/5 of the height of the container.

When the container is tipped on its side, so its height is 5 cm, the depth will still be 4/5 of the height.

  (4/5)(5 cm) = 4 cm

The depth will be 4 cm.

The parent council is in charge of making lemonade for field day.They purchased 19 bags of lemon.each bag has 24 lemons.The recipe says that a gallon of lemonade will require 8 lemons.they will be able to pour 12 cups of lemonade from each gallon that they make.How many cups of lemonade will the parent council be able to serve?

Answers

Answer:

684

Step-by-step explanation:

19 x 24 = 456    

456 divided by 8 = 57

57 x 12 = 684

Ms. Davis writes an equation on the board.
(-6) + 6 = 0
Which scenario could represent this equation?

Answers

There are many answers we could go with here. One such answer could be the following:

You are 6 meters below the ocean surface doing a deep sea dive. This is represented by -6. Then going up 6 meters means you'll add on 6 to say (-6)+6 = 0, showing that you are at the 0 meter height, aka sea level.

---------------------------------

Another possible answer could be this:

You are 6 dollars in debt. Debt is represented with negative numbers. You then somehow get 6 dollars in which you use to pay off that debt. Your balance is now (-6) + 6 = 0 dollars.

---------------------------------

Another possible answer could be this:

Walking to the left is defined as moving in a negative direction. So -6 could mean we've moved 6 feet to the left. On the flip side, +6 means we move 6 feet to the right. The two cancel each other out (-6)+6 = 0 which says at the end of the day, we haven't moved anywhere. In other words, our displacement is 0 feet.

-----------------------------------

I'm sure there are other more creative ways to answer this question, but those three ideas above are good possible starting points.

PLZZZ help!!!
Find the area and the perimeter of the shaded regions below. Give your answer as a completely simplified exact value in terms of π (no approximations). The figures below are based on semicircles or quarter circles and problems b), c), and d) are involving portions of a square.

Answers

Answer:

[tex]A_s = 36\pi - 72[/tex]

Step-by-step explanation:

[tex]A_{shaded} = \frac{A_{circle}}{4} - A_{triangle}\\\\A_c = \pi r^2\\A_c = \pi(12)^2\\A_c = 144\pi\\\\A_t = \frac{b * h}{2} \\\\A_t = \frac{12 * 12}{2} \\\\A_t = 72\\\\\\A_s = \frac{144\pi }{4} - 72\\A_s = 36\pi - 72[/tex]

Is -√15 an imaginary or irrational number?

Answers

Answer:

irrational

[tex]\sqrt{-15}[/tex] would be imaginary

Step-by-step explanation:

Hey there! I'm happy to help!

An irrational number is a real number that can't be written as a ratio of two other integers (non-fraction numbers) so a number like π or √2.

An imaginary number is one that breaks a mathematical rule so it can't necessarily exist as a real number. The main imaginary number is i, and here is the thing it does that breaks the rules and makes it imaginary:

i²=-1

so

√-1=i

We know that you cannot square a real number and have it be negative because a negative number can only be obtained by multiplying a positive and a negative number. This also shows us that there cannot be a square root of a negative number.

We have the number -√15. The placement of that negative sign is CRUCIAL! The square root of any positive integer is going to be a real number, but the square root of a negative number is nonexistent aka imaginary. √-15 would be an imaginary number, but we have -√15, which is the same thing as -1·√15. The negative sign is not inside the square root, which makes it not imaginary, so this is an irrational number. You can plug it into a calculator and it would be -3.872983346....

Have a wonderful day and keep on learning :)

what's 700.00divided by 120​

Answers

The answer is 3.5 because 120 can go into 700 three 1/2 times

Answer:

[tex]\Large \boxed{\frac{35}{6} }[/tex]

Step-by-step explanation:

[tex]\displaystyle \frac{700}{120}[/tex]

Reduce and simplify the fraction to lowest terms.

[tex]\displaystyle \frac{20(35)}{20(6)}[/tex]

[tex]\displaystyle \frac{35}{6}[/tex]

1. Suzette ran and biked for a total of 80 miles in 9 hours. Her average running speed was 5 miles per hour (mph) and her average biking speed was 12 mph. Let x = total hours Suzette ran. Let y = total hours Suzette biked. Use substitution to solve for x and y. Show your work. Check your solution. (a) How many hours did Suzette run? (b) How many hours did she bike?

Answers

Answer:

a) Suzette ran for 4 hours

b) Suzette biked for 5 hours

Step-by-step explanation:

Speed is rate of distance traveled, it is the ratio of distance traveled to time taken. It is given by:

Speed = distance / time

The total distance ran and biked by Suzette (d) = 80 miles, while the total time ran and biked by Suzette (t) = 9 hours.

For running:

Her speed was 5 miles per hour, let the total hours Suzette ran be x and the total distance she ran be p, hence since Speed = distance / time, therefore:

5 = p / x

p = 5x

For biking:

Her speed was 12 miles per hour, let the total hours Suzette ran be y and the total distance she ran be q, hence since Speed = distance / time, therefore:

12 = q / y

q = 12y

The total distance ran and biked by Suzette (d) = Distance biked + distance ran

d = p + q

80 = p + q

80 = 5x + 12y                 (1)

The total time taken to run and bike by Suzette (t) = time spent to bike + time spent to run

t = x + y

9 = x + y                         (2)

Solving equation 1 and equation 2, multiply equation 2 by 5 and subtract from equation 1:

7y = 35

y = 35/7

y = 5 hours

Put y = 5 in equation 2:

9 = x + 5

x = 9 -5

x = 4 hours

a) Suzette ran for 4 hours

b) Suzette biked for 5 hours

Can integers be written as fractions?

Answers

Answer:

Step-by-step explanation:

Yes you can just write them with denominator 1,

So 3 = 3/1 and -6 = -6/1.

Answer:

Yes.

Step-by-step explanation:

ALL real numbers can be written as fractions, and since integers fall under the category of real numbers, it is official that they can be written as fractions.

I am joyous to assist you at any time.

A rectangle's perimeter is equal to 27 plus its width. The length of the rectangle is four times its width. What is the
width of the rectangle in units?

Answers

Answer:

P = 3

Step-by-step explanation:

L = 4*w

P = 27 + w

P = 2*L + 2*w = 27 + w

2L = 2*(4w) = 8w

Put that into the perimeter formula (the first one.)

P = 8w + 2w = 27 + w

10w = 27 + w                        Subtract w from both sides

9w = 27                                Divide by 9

w = 27 / 9                          

w = 3

You should find the Length just so you can check it.

L = 4w

L = 4*3

L = 12

Check

P = 2L + 2w

P = 2*12 + 2*3 = 30

P = 27 + w

P = 27 + 3

P = 30 and everything checks.

Tim owns a clothing store where he designs pairs of shorts, s, and T-shirts, t.
He sells the shorts for $12 and the T-shirts for $8 each. Tim can work 18
hours a day, at most. It takes him 30 minutes to design a T-shirt and 45
minutes to design a pair of shorts. He must design at least 12 items each
day, but he cannot design more than 30 items in one day. Which set of
inequalities below represents this scenario?
A. s 2 12 + $ ss 30 + tiss 24-0.66t, s2; 0; t2 0
B. s> 12-tss 30 - t Ss 24 -0.66t, s 2 0; t> 0
O c. s? 12-ts? 30-tss 24 -0.66t, s 2 0;t20
D. S 2 12-tss 30-ts 24 -0.66t, s 20; t 0​

Answers

Answer:

The correct option is;

B. s ≥ 12 - t, s ≤ 30 - t, s ≤ 24 - 0.66·t

Step-by-step explanation:

The given parameters are;

The number of T-shirts, t, and shorts, s, Tim must design a day = 12

The maximum number of T-shirts and shorts Tim can design a day = 30

The maximum number of hours Tim can work = 18 hours

Therefore, we have;

The number of shorts Tim designs in a day is ≥ The minimum number of T-shirts and shorts Tim must design a day less the number of T-shirts Tim designs

Which gives;

s ≥ 12 - t

Also the number of shorts Tim designs in a day is ≤ The maximum number of T-shirts, and shorts, Tim can design a day less the number of T-shirts Tim designs

Which gives;

s ≤ 30 - t

The number of 45 minute period for the design of shorts in 18 hours = 18×60/45 = 24

The fraction of 36 minutes in 45 minutes = 36/45 = 0.667

Therefore we have;

The number of shorts Tim designs in a day is ≤ The number of 45 minute periods in 18 hours less the number of 36 minutes periods used to design T-shirts

Which gives;

s ≤ 24 - 0.66·t

The correct option is s ≥ 12 - t, s ≤ 30 - t, s ≤ 24 - 0.66·t.

Answer:

B. s> 12-tss 30 - t Ss 24 -0.66t, s 2 0; t> 0

Step-by-step explanation:

Hope this helps!!

Given that the trinomial x^2+ 11x + 28 has a factor of x +4, what is the other factor?

Answers

Answer:

the other factor is (x+7)

Step-by-step explanation:

Given x^2+11x+28

factor into

x^2+7x + 4x + 28

=x(x+7) + 4(x+7)

= (x+4)(x+7)

Answer: the other factor is (x+7)

Christine's gross monthly salary at her job
is $5,250. She has the following
deductions from her paycheck.
What is Christine's net take-home pay per
month?

Answers

Answer:

  $2587.87 per month

Step-by-step explanation:

The listed deductions are ...

25% withheld for federal income tax9.3% withheld for California state income tax6.2% withheld for Social Security tax1.45% withheld for Medicare Tax0.9% withheld for SDI- Disability Insurance5% goes into her retirement 401K account$150 goes to health insurance/ dental for her family

The percentages have a total of ...

  25 +9.3 +6.2 +1.45 +0.9 +5 = 47.85 . . . percent

So, Christine's take-home pay is ...

  $5250(1 -0.4785) -150 = $2587.87 . . . per month

A farmer has 2000 plants. He wants to plant these in such a way that the number of rows and the number of columns remain same. Find the minimum number of plants he needs more for this. 4.

Answers

Answer:

500 plants

Step-by-step explanation:

Step one:

given that the number of plants the farmer has is 2000

for him to plant in equal rows and column he needs an amount of plant the is a perfect square.

The closest number to a perfect square is 2500

Step two:

From the above figure which is 2500 plants, the farmer is able to plant 50 rows and 50 columns.

multiplying these values together will amount to 2500

Hence the farmer needs at least 500 plants to achieve this

Answer:

25

Step-by-step explanation:

Number of plants = 2000

He wants to plant such that number of rows and column of plants remain the same, Find the minimum number of plants he needs more for this:

To obtain this,

We take the square root of 2000

Sqrt(2000) = 44.721359

Therefore, the number of columns and rows we need is a natural number whereby the square root of its square is perfect (that is leaves no remainder).

The minimum number will thus be ; the next natural number or integer after 44.721359, whish is 45

Taking the square of 45

45^2 = 2025

The minimum number of plants needed more is :

2025 - 2000 = 25 plants

I need help ASAP!! Please explain how to solve the problem

Answers

Answer:

area of base = pi x radius^2 = (ill use 3.14 as pi)

3.14 x 7^2 = 153.86 in^2

height = 12

circunference of base = 2 x pi x radius or pi x diameter (diameter = 2 x radius) = 3.14 x 14 = 43.96 in

surface area = perimeter of base x height (lateral area) + 2 x area of base

= 43.96 x 12 = 527.52 + 2(153.86) = 835.24

hope that answers your question

(be careful, i generally use 3.14 as pi, but if your question says to use other forms as pi, then just replace pi as what the question asks, ive given you the formulas on how to calculate area and circumf., so i hope it should be easier...)

the sum of two numbers is 63. one ninth of the first number plus one sixth of the second number is 21. find the numbers

Answers

Answer: x= -189, y =252

Step-by-step explanation:

Let the first number be x and second number be y

so

x + y = 63

x = 63-y

now

1/9 of x + 1/6 of y = 21

x/9 + y/6 = 21

substituting x's value from equation i

(63-y)/9 + y/6 = 21

(378-6y+9y)/54 = 21

378+3y = 1134

3y = 1134-378

so, 3y = 756

so, y = 756/3

so, y = 252

now

x = 63-252

so, x = -189

A wooden jewelry box has the shape of a prism with a regular hexagonal base of 85.3 in2. The sides of the hexagonal base are all 5.73 inches. If the height of the box is 18.10 inches, what is the surface area of the wood used to make the jewelry box?

Answers

Answer:

792.9 in²

Step-by-step Explanation:

Given:

Area of the base of the regular hexagonal prism box (B) = 85.3 in²

Each side length of hexagonal base (s) = 5.73 in

Height of prism box (h) = 18.10 in

Required:

Surface area of the wood used in making the hexagonal prism box

SOLUTION:

Surface area for any given regular prism can be calculated using the following formula: (Perimeter of Base × height of prism) + 2(Base Area)

Perimeter of the hexagonal base of the prism box = 6(5.73) (Note: hexagon has 6 sides.)

Perimeter of base = 34.38 in

Height = 18.10 in

Base area is already given as 85.3 in²

Surface area of the hexagonal prism box [tex] = (34.38*18.10) + 2(85.3) [/tex]

[tex] = 622.278 + 170.6 = 792.878 in^2 [/tex]

Surface area of the wood used in making the jewelry box ≈ 792.9 in²

i’m pretty bad at math

Answers

Answer:

2m - 3

Step-by-step explanation:

We know that Jim ran m miles. Twice that amount is simply 2 times that, or 2 * m which simplifies to 2m. 3 miles fewer means that we have to subtract 3 from that quantity, so the answer is 2m - 3.

Answer:

2m - 3

Step-by-step explanation:

Distance ran by Jim = m

Twice as far as Jim : 2 * m = 2m

3 miles fewer than '2m' = 2m - 3

Distance ran by Kelly = 2m - 3

We use the formula V = πr2h to find the volume of a cylinder. Make h the subject of the formula.

Answers

Answer:

V / ( pi r^2)  = h

Step-by-step explanation:

V = pi r^2 h

Divide each side by pi r^2

V / ( pi r^2)  = pi r^2 h / pi r^2

V / ( pi r^2)  = h

En una empresa trabajan 60 personas. Usan gafas el 16% de los hombres y el 20% de las mujeres. Si el número total de personas que usan gafas es 11. ¿Cuántos hombres y mujeres hay en la empresa?

Answers

Pregunta completa:

En una empresa trabajan 60 personas. Usan gafas el 16% de los hombres y el 20% de las mujeres. Si el numero total de personas que usan gafas es 11. ¿Cuantos hombres y mujeres hay en la empresa?

Responder:

Hombres = 25

Mujeres = 35

Explicación paso a paso:

Dado lo siguiente:

Número de personas que trabajan en la empresa = 60

Porcentaje de hombres que usan anteojos = 16%

Porcentaje de mujeres que usan anteojos = 20%

Número total de personas que usan anteojos = 11

Suponga, Número de hombres en la empresa = m

Número de mujeres = número total - número de hombres = 60 - m

Por lo tanto,

16% de los hombres = 0,16 m

20% de mujeres = 0,2 (60 - m) = 12 - 0,2 m

Por lo tanto,

0,16 m + 12 - 0,2 m = 11

- 0,04 m = 11 - 12

-0,04 m = - 1

m = 1 / 0.04 = 25

Por tanto, Número de hombres en la empresa = m = 25

Número de mujeres en la empresa = (60 - m) = (60 - 25) = 35 mujeres

What is 9x - 2 - 13x + 16 simplified ???

Answers

Answer:

Simplifying

9x + -2 + -13x + 16 = 0

Reorder the terms:

-2 + 16 + 9x + -13x = 0

Combine like terms: -2 + 16 = 14

14 + 9x + -13x = 0

Combine like terms: 9x + -13x = -4x

14 + -4x = 0

Solving

14 + -4x = 0

Solving for variable 'x'.

Move all terms containing x to the left, all other terms to the right.

Add '-14' to each side of the equation.

14 + -14 + -4x = 0 + -14

Combine like terms: 14 + -14 = 0

0 + -4x = 0 + -14

-4x = 0 + -14

Combine like terms: 0 + -14 = -14

-4x = -14

Divide each side by '-4'.

x = 3.5

Simplifying

x = 3.5

Step-by-step explanation:

Answer:

-4x+14

Step-by-step explanation:

Collect "Like Terms" (9x and -13) = -4x

-4x - 2 + 16= -4x + 14

what is the value of 600.79-40.0032+5.01 to the nearest Hundredths​

Answers

Answer:

565.80

Step-by-step explanation:

600.79-40.0032+5.01 =  565.7968

565.7968 to the nearest Hundredths​ =

565.80

Answer:

565.80

Step-by-step explanation:

600.79 - 40.0032 + 5.01

600.79 - 40.0032 = 560.7868

560.7868 + 5.01 = 565.7968

565.7968 to the nearest hundredth = 565.80

18 m
6 m
6 m
16 m
16 m
14 m

Answers

16 m this is the answer because i don’t know what you’re asking so i’m just guessing, sorry

Find the area of a triangle with legs that are: 16 m, 12 m, and 8 m.
O A. 16.4 m2
B. 54 m2
C. 46.5 m2
D. 38.2 m2

Answers

ans. 46.5m2......................

The grades on the last art exam had a mean of 75%. Assume the population of grades on art exams is known to be distributed normally, with a standard deviation of 7%. Using the empirical rule, approximately what percent of students earn a score between 75% and 82%? 68% 16% 32% 34%

Answers

Find the z-score for both 75% and 82% then subtract them.

Z-score for 75%:

75 -75 /7 = 0-7 = 0

Using empirical rule, This translates to a percentile if 50%

Z-score for 82%

82-75 /7 = 7/7 = 1

Using empirical rule, this translates to 84.13%

Answer = 84.13-50 = 34.13% round to 34%

Answer:

34%

Step-by-step explanation:

Well 75+7=82 and one hop on the graph is 34%.

Can you help me please?

Answers

Answer: Repeating decimals

Fractions

Terminating decimals.

Step-by-step explanation:

A rational number is a number that can be expressed as [tex]\dfrac{p}{q}[/tex] , where 'p' and 'q' are two integers and 'q' is not equal to 0.The set of rational number includes integers (...-1,0,1....), terminating decimals, and repeating decimals and fractions.

But imaginary numbers comes in the set of irrational number as it cannot be expressed as fractions.

Hence, the correct choices are :

Repeating decimals

Fractions

Terminating decimals.

Factor 6x2 + 12x - 9 completely.
A
3(2x² + (-3)
+ 4x –
B 6x(x – 7)
3(2x - 3)(x + 1)
D 3(2x + 1) (x – 3)

Answers

Answer:

work is pictured and shown

Need help asap algebra 2 math

Answers

life hack: if you own a TI-84 graphing calculator you can just input that into the calculator on the graphing part and then you see which equation’s graph matches the graph you were given in the problem.
Answer: Choice C

f(x) = 5x^2 + 25x + 30

============================================================

Explanation:

The roots, aka x intercepts, of this curve are x = -3 and x =  -2. This is where the graph crosses the x axis.

Since x = -3 is a root, this makes x+3 a factor of the quadratic. Similarly, x = -2 leads to x+2 as another factor. I'm using the zero product property.

So far we have found that the polynomial is (x+3)(x+2). This isn't the full factorization because if we plugged x = -1 into that expression, then we would get

y = (x+3)(x+2)

y = (-1+3)(-1+2)

y = (2)(1)

y = 2

But we want y = 10 instead. So we must multiply that factorization by 5 to jump from 2 to 10 (i.e. 5*2 = 10)

Therefore, the full factorization of this parabola is y = 5(x+3)(x+2)

Now let's expand everything out and simplify

y = 5(x+3)(x+2)

y = 5(x^2+2x+3x+6)

y = 5(x^2+5x+6)

y = 5x^2+5*5x+5*6

y = 5x^2 + 25x + 30

Choice C is the final answer

-------------------------

To check this, we can plug in x = -3 and we should get 0

y = 5x^2 + 25x + 30

y = 5(-3)^2 + 25(-3) + 30

y = 5(9) + 25(-3) + 30

y = 45 - 75 + 30

y = -30 + 30

y = 0

This proves that x = -3 is a root of y = 5x^2 + 25x + 30

I'll let you check x = -2. You should also get y = 0 when plugging this x value in.

Plugging x = -1 should lead to y = 10 as the last bit of confirmation. I'll let you check this one as well.

Other Questions
12. '' ? 1. 2. 3. 4. fast pls "Let's assume that the government decides to regulate a natural monopoly by forcing them to produce at a point where the natural monopoly's demand curve intersects average cost. In this case, the price will __________ and the quantity will ________ when compared to the natural monopoly if it were allowed to operate unregulated." hello, can i please get some please on this one ASAP! i will mark as BRAINLIEST!! 3.7500 x 104 + 9.7100 x 105 Compare and contrast a perceptual region and a formal region What is the equation of the line of best fit for the following data? Round theslope and y-intercept of the line to three decimal places. There is shortage of water in your community.Write a letter to your D.C.E or M.C.E suggest three way this problem can be fixed verbo formado con estas letras cerrienquer What is the value of m in the figure below? If necessary, round your answer tothe nearest tenth of a unit. Anh/Ch hy chng minh: Hc thuyt hnh thi kinh t - x hi l hn tng ca ch ngha duy vt lch s; cn s vn ng pht trin ca x hi loi ngi tri qua cc hnh thi kinh t - x hi l mt qu trnh lch s - t nhin. Rules of replacement are acceptable because the rules capture an important idea about logical equivalence.a) trueb) false PLS HELP ME ON THIS QUESTION I WILL MARK YOU AS BRAINLIEST IF YOU KNOW THE ANSWER PLS GIVE ME A STEP BY STEP EXPLANATION!!Paula ran a total of 12 miles over the course of 2 track practices. What is the unit rate?A. 6 milesB. 6 miles per practiceC. 4 miles per practiceD. 10 miles per practice How are reactivity and electronegativity related? 1 point). An element will be more reactive if it has a very high or very low electronegativityElements with lower electronegativity are more reactive than elements with higher electronegativityElements with higher electronegativity are more reactive than elements with lower electronegativityAnmentess reactive it has a very high or very low electronegativity what is the solution to the equation x = 16? Tina's age is 4 years less than 3 times her niece's age. If her niece's age is x years, which of the following expressions best shows Tina's age? x 4 4x 3 3x 4 4 3x how to find y in this equation 11=12-14y The digits 0,1,2,3,4,5 and 6 are used to make 3 digit codesIn case where digits may be repeated, how many codes are numbers that are greater than 300 and exactly divisible by 5? It is often possible to change a hydrate into an anhydrous compound by heating it to drive off the water (dehydration). A 43.19 gram sample of a hydrate of MgBr2 was heated thoroughly in a porcelain crucible, until its weight remained constant. After heating, 27.21 grams of the anhydrous compound remained. What is the formula of the hydrate? help me please pelase please Find the equation of the line with a slope of 1/2 through (4, 5)